Как получить явный вид функции Грина уравнения Клейна-Гордона?

Определение функции Грина для уравнения Клейна-Гордона гласит:

( т 2 2 + м 2 ) г ( Икс , т ) "=" дельта ( т ) дельта ( Икс )
По этим ресурсам:

  1. Функция Грина для неоднородного уравнения Клейна-Гордона , функция Грина выглядит так:

г ( Икс , т ) "=" θ ( т ) 2 π дельта ( т 2 | Икс | 2 ) м 2 π θ ( т | Икс | ) Дж 1 ( м т 2 | Икс | 2 ) м т 2 | Икс | 2

  1. Из википедии есть несколько видов пропагатора , что зависит от выбора контура. Они очень похожи на вышеупомянутые, но все же не идентичны мне. Например, функция продвинутого и отсталого Грина очень похожа на приведенное выше выражение.

Я хочу получить явную форму указанной выше функции зеленого цвета , вот что я пробовал:

Предполагать:

г ( Икс , т ) "="   г 3 п   г ю е я ( п Икс ю т ) г ( п , ю )

Подставляя уравнение Клейна-Гордона, получаем:

г ( п , ю ) "=" 1 ( 2 π ) 4 1 ю 2 п 2 м 2

Затем:

г ( Икс , т ) "=" 1 ( 2 π ) 4   г 3 п   г ю е я ( п Икс ю т ) 1 ю 2 п 2 м 2

В приведенной выше формуле полюс находится на реальной оси, чтобы получить окончательный ответ, нужно немного отклонить полюс от реальной оси. По выбору Фейнмана левый полюс находится немного выше, а правый полюс немного ниже, вот так:введите описание изображения здесь

Тогда у нас есть:

г ( Икс , т ) "=" 1 ( 2 π ) 4   г 3 п   г ю е я ( п Икс ю т ) 1 ю 2 п 2 м 2 + я ϵ интегрировать  ю "=" θ ( т ) я ( 2 π ) 3   г 3 п е я п Икс е я п 2 + м 2 т 2 п 2 + м 2 + θ ( т ) я ( 2 π ) 3   г 3 п е я п Икс е я п 2 + м 2 т 2 п 2 + м 2 интегрировать  ф  и  θ "=" я ( 2 π ) 2 θ ( т ) | Икс | 0 п грех ( п | Икс | ) е я п 2 + м 2 т п 2 + м 2 + я ( 2 π ) 2 θ ( т ) | Икс | 0 п грех ( п | Икс | ) е я п 2 + м 2 т п 2 + м 2

Приведенный выше вывод кажется безошибочным, и я не знаю, как продолжить п интеграл, и я не вижу сходства текущей формулы с закрытой формулой, приведенной в ресурсах 1 и 2.

Интегрировать п , я обнаружил, что этот интеграл из книги может помочь:

0 е β γ 2 + Икс 2 потому что б Икс "=" β γ γ 2 + β 2 К 1 ( γ β 2 + б 2 ) ,  с:  р е β > 0 , р е γ > 0

Редактировать2

--- чтобы обратиться к обновленному ответу @Solenodon Paradoxus

В ответе предлагалось повернуть контур против часовой стрелки ( ю "=" я ю ), поэтому:

г Ф ( п , ю ) "=" 1 ю 2 | п | 2 м 2 + я ϵ "=" 0 г л е л ( ю 2 + | п | 2 + м 2 я ϵ )

Подключите выше г Ф ( п , ю ) в интеграл четырех импульсов:

г Ф ( Икс , т ) "=" я ( 2 π ) 4 0   г л е ( м 2 я ϵ ) л г ю г 3 п е л ю 2 + т ю е л п 2 + я Икс п "=" я 16 π 2 0   г л е ( м 2 я ϵ ) л 1 л 2 е т 2 4 л  с  т 2 "=" т 2 | Икс | 2

С этого момента ϵ Неважно проводить приведенный выше интеграл, просто опустим его.

г Ф ( Икс , т ) "=" я 16 π 2 0 г л е м 2 л 1 л 2 е т 2 4 л "=" я м 2 16 π 2 0 г ξ е ξ е м 2 т 2 4 ξ 1 ξ 2
Когда т 2 > 0 , подынтегральная функция расходится в ξ 0 , интеграл не может быть проведен.

Когда т 2 < 0 , у нас есть:

я м 4 π 2 т 2 К 1 ( м т 2 )
где К 1 является модифицированной функцией Бесселя, это в точности форма пропагатора Фейнмана, когда т 2 < 0 .

Вопрос: Хотя наш результат торжествует с одной стороны, как насчет другой стороны( т 2 > 0 )? Как мы можем получить его из описанной выше процедуры? На каком шаге мы исключили эту возможность?

Ответы (2)

Функции Грина не уникальны. Любое решение, которое удовлетворяет однородному уравнению,

( т 2 2 + м 2 ) ф "=" 0
в интересующей области можно добавить к функции Грина без искажения неоднородного уравнения. Поэтому однородная часть должна быть задана граничными условиями. Вы можете получить часть функции Бесселя версии функции Грина Фейнмана, найдя стационарную функцию Грина с 4-мерным пространством, то есть:
( 1 р 3 р [ р 3 р ] + м 2 ) г ( р ) "=" я Ом 4 р 3 дельта ( р )
и аналитически продолжается до мнимого времени. Внутри переднего светового конуса, где аргумент К является мнимым, дает функцию Ганкеля первого рода:
м 4 π 2 т 2 К 1 ( я м т 2 ) "=" м 8 π т 2 ЧАС 1 ( 2 ) ( м т 2 ) "=" м 8 π т 2 [ Дж 1 ( м т 2 ) я Д 1 ( м т 2 ) ] ,
с Дж а и Д а функции Бесселя первого и второго рода соответственно.

Поскольку и оператор, и неоднородная часть вещественны, мнимая часть функции Грина должна быть решением однородного уравнения, а действительная часть должна решать неоднородное. То есть:

( т 2 2 + м 2 ) Я { м 4 π 2 т 2 К 1 ( я м т 2 ) } "=" 0 ,   а н г ( т 2 2 + м 2 ) Ре { м 4 π 2 т 2 К 1 ( я м т 2 ) } "=" дельта 4 ( Икс ) ,
поэтому функция Грина является реальной частью этого выражения.

Особенности, которые еще предстоит показать в этом месте: что разделение на запаздывающие и продвинутые пропагаторы действительно (необходимо для сохранения причинно-следственной связи), а предел нулевой массы пропагатора дает правильный предел. Энергию, вводимую импульсом, нельзя отследить, потому что она бесконечна.

Возможно, второе уравнение следует записать так:
[ 1 р 3 р р 3 р + м 2 ] г ( р ) "=" дельта ( р )
?
Хороший улов на неуместных скобках. Однако правая сторона нуждалась только в исправлении знака. Это 4-мерная дельта-функция, поэтому она должна иметь 4-мерный телесный угол и коэффициент р 3 .
Я все еще думаю, что правая часть должна измениться, исходное уравнение:
( т 2 2 + м 2 ) г ( Икс , т ) "=" дельта ( т ) дельта ( Икс )
Вы хотите произвести замену т я ш , приведенное выше уравнение принимает вид:
( 4 2 + м 2 ) г ( р ) "=" дельта ( я ж ) дельта ( Икс )
Сейчас 4 2 "=" 1 р 3 р ( р 3 р ) + л 2 р 2 , где л 2 — оператор, дифференцирующий угловую часть. Если мы предположим г ( р ) "=" г ( р ) , л 2 действие на него просто дает ноль. Таким образом, уравнение принимает вид:
( 1 р 3 р ( р 3 р ) + м 2 ) г ( р ) "=" дельта ( я ж ) дельта ( Икс )
Вопрос , на данном этапе, я думаю, мы рассматриваем ш быть реальной переменной. Однако, когда доходит до дельта ( я ш ) (хорошо определенный?), есть ли у нас тождество, которое дельта ( я ш ) "=" я дельта ( ш ) ? поэтому правая сторона, вероятно, должна быть изменена как я дельта ( р ) ?
Действительно, я пропустил я при переходе к евклидову 4-мерному пространству. И да, у нас есть дельта ( я ш ) "=" я дельта ( ш ) .
Как насчет первого комментария, что я думаю 1 р 3 Ом 4 не нужно.
Ах, эта часть неверна. Дельта-функция должна подчиняться тождеству:
дельта ( Икс ) г В "=" 1 ,
независимо от размерности Икс . Это означает, что когда вы меняете переменные, например, на какую-то сферическую систему координат, дельта-функция должна быть разделена на определитель якобиана , чтобы исключить фактор, связанный с элементом объема. В этом случае, г В "=" Ом 4 р 3 г р .
Я разместил полное доказательство в виде вопросов и ответов: physics.stackexchange.com/questions/280212/… .

Вы можете попробовать использовать метод правильного времени:

1 п 2 м 2 + я ε "=" 0 г л опыт [ л ( п 2 м 2 + я ε ) ] .

Хитрость заключается в том, чтобы сначала выполнить (гауссовский) интеграл импульса, а затем перейти к интегралу по л . Это должно дать функцию Бесселя. Дайте мне знать в комментариях, если у вас есть дополнительные вопросы.

ОБНОВЛЕНИЕ: Как обращаться с подписью Лоренца в распространителе?

Нас интересует следующий интеграл:

Д ( Икс ) "=" г 4 п ( 2 π ) 4 е я п мю Икс мю п 2 м 2 + я ε "=" г ю 2 π г 3 п ( 2 π ) 3 е я ю т е я п Икс ю 2 п 2 м 2 + я ε .

Теперь мы можем повернуть контур интегрирования в комплексной плоскости так, чтобы через него не проходил ни один полюс. Я поверну контур вашего исходного поста на 90 градусов против часовой стрелки:

Д ( Икс ) "=" я г ю 2 π г 3 п ( 2 π ) 3 е ю т е я п Икс ю 2 + п 2 + м 2 .

Я также перешел к новой переменной интеграции: ю "=" я ю . Теперь знаменатель всегда положителен, и вы можете использовать метод правильного времени. Все интегралы будут гауссовыми и сходящимися.

ОБНОВЛЕНИЕ 2 : я ε позволяет нам вращать контур против часовой стрелки, но запрещает другое (аналогичное) преобразование, например, вращение его по часовой стрелке, как это сделали вы. Это связано с тем, что ни один полюс не должен пересекать контур во время его деформации. Следовательно, ю "=" я ю , нет я ю . Гладкие преобразования контура не меняют интеграл, пока через него не проходит ни один полюс.

Итак, роль, которую сыграл я ε состоит в том, чтобы определить, как интеграл будет преобразован в евклидов интеграл. В евклидовом случае (после поворота контура) можно опустить я ε .

ОБНОВЛЕНИЕ 3: так как мы повернули контур, ю становится воображаемым и ю "=" я ю действительна (собственно, весь смысл введения ω′). Следовательно, ю 2 + п 2 + м 2 не может быть меньше нуля!

ОБНОВЛЕНИЕ 4: что касается вашего окончательного ответа, я подозреваю, что если вы возьмете ю "=" я ю и не я ю (вам нельзя проходить через столб, помните?) тогда вы получите правильный ответ. Я не уверен, конечно, но я бы так и поступил.

Вы получаете различные функции Грина. На самом деле все зависит от того, как контур обходит полюса. Или где подключить я ε , если хотите.

ОБНОВЛЕНИЕ 5: Как обращаться с т 2 > 0 случай?

Ну, вот что я придумал. Вы можете вращать контуры трех интегралов по п вместо того, что закончилось ю . После аналогичных вычислений вы придете ко второй функции Ганкеля ЧАС 1 ( 2 ) . В качестве альтернативы, поскольку мы везде используем комплексные числа, мы могли бы просто сделать аналитическое продолжение результата для т 2 < 0 , что дало бы нам точно такую ​​же функцию Ганкеля.

Я все еще пытаюсь найти четкое объяснение, почему появляется дополнительный дельта-символ (не то чтобы он не мог появиться, так как мы только вычислили наш пропагатор для т 2 > 0 и т 2 < 0 к настоящему времени).

Вы предполагали, что п 2 м 2 > 0 ? Если написано как у вас, при выполнении интеграла по импульсу коэффициенты перед п 2 должна быть по крайней мере отрицательной в действительной части.
с тех пор ю 2 п 2 всегда сохраняются, интегрировать четыре импульса кажется невозможным с помощью интеграла Гаусса.
@buzhidao в моих обозначениях, п 2 "=" п 0 2 п 2 . Нет, п 2 м 2 не всегда больше нуля. Можно использовать аналитическое продолжение гауссовского интеграла.
См. мой первый комментарий, я думаю, что интеграл Гаусса может быть правильно определен только в том случае, если действительная часть коэффициента квадрата меньше нуля. С другой стороны, вы просто не можете этого сделать, это расходится.
@buzhidao, если вы рассматриваете евклидов случай, (евклидов) пропагатор пропорционален ( п 2 + м 2 ) 1 и все интегралы Гаусса определены корректно. Затем вы можете получить лоренцев пропагатор с помощью аналитического продолжения (переходя к мнимым временным интервалам), как вы объяснили в своем вопросе. В качестве альтернативы вы можете думать, что интегралы Гаусса определяются аналитическим продолжением, даже если коэффициент перед квадратным членом положителен.
@buzhidao Я обновил свой ответ, добавив подробности о том, как правильно обращаться с подписью Лоренца, пожалуйста, посмотрите. Поскольку это вопрос домашнего задания и упражнений, я не буду приводить полный расчет. Вы должны будете выполнить детали расчета самостоятельно, извините. Если у вас есть еще вопросы, пожалуйста, дайте мне знать.
Я добавляю вывод и перечисляю свои проблемы, пожалуйста, посмотрите.
@buzhidao Я еще раз обновил свой ответ, пожалуйста, посмотрите.
Ваше объяснение вполне понятно, я неправильно понял деформацию контура как замену переменных, поэтому приводит к беспокойству по поводу проблемы расходимости метода собственного времени. Теперь я обновил расчет и поднял новую проблему, пожалуйста, посмотрите.
@buzhidao Мне придется подумать об этом. Свяжутся в течение 1-2 дней.
@buzhidao Пока я думаю об этом, взгляните, пожалуйста, на мое обновление № 5.